sexta-feira, 6 de julho de 2012

053-Limite com Fatorial e Exponencial Geral

É perceptível que, para um inteiro [;n>1;], temos [;n^n > n!;]. Sabe-se que [;\lim_{n \rightarrow \infty} k^{1/n}=1;] e [;\lim_{n \rightarrow \infty} n^{1/n}=1;] . Mas, qual o valor de

 [;\lim_{n \rightarrow \infty}\left(\frac{n!}{n^n}\right)^{1/n};]?

Para investigar este último limite, analisaremos a área sob o gráfico da função [;f(n)=ln(n);], primeiramente no intervalo [;[2,4];] e, a partir deste, generalizaremos o estudo para [;[1,n];].


Observem no diagrama que a área [;S_2^4;] no intervalo [;[2,4];] sob a curva descrita pela função definida por [;f(n)=ln(n);] e limitada pelo eixo [;On;]é maior que a a soma das áreas dos retângulos menores [;BEFC;]e [;DFLJ;] e menor que a soma das áreas dos retângulos maiores [;AEFD;]e [;GFLH;].

Assim,
[;S_{BEFC};]+ [;S_{DFLJ};] [;<;] [;S_2^4;] [;<;] +[;S_{AEFD};]+ [;S_{GFLH};] [;\Rightarrow;]

[;1.ln(2)+1.ln(3)< \int_2^4 ln(n) < 1.ln(3)+1.ln(4) \Rightarrow;]
 [;1.ln(2)+1.ln(3)+...ln(n-1)<\int_2^{n} ln(n)<1.ln(3)+1.ln(4)+1.ln(n) \Rightarrow;]
 [;ln [(n-1)!] < \left[nln(n)-n \right] _ 2^n<ln \left[\frac{n!}{2}\right];], para [;n \geq 2;]

Mas como [;ln[(n!)]>ln \left[\frac{(n!)}{2}\right];], também podemos fazer

[;ln [(n-1)!] \leq \left[nln(n)-n \right] _ 1^n \leq ln [(n!)];], para [;n\geq 1;]

Desenvolvendo a integral definida, passando para exponencial e cancelando as bases, temos

 [;ln [(n-1)!] \leq ln(n^n)-n-(-1) \leq ln [n!] \Rightarrow;]

[;e^{ln[(n-1)!]} \leq e^{ln(n^n)-n+1} \leq e^{ln[n!]} \Rightarrow;]

[;(n-1)! \leq n^ne^{-n}.e \leq n!;]

Agora vamos analisar esta desigualdade considerando cada membro. 


[;I;]) [;n^ne^{-n}.e \leq n!;] 

Extraindo a raíz enésima,

 [;ne^{-1}e^{1/n} \leq (n!)^{1/n};]

Dividindo por [;n;]

 [;\rightarrow \frac{1}{e}.e^{1/n} \leq \left(\frac{n!}{n^n}\right)^{1/n};]


[;II;]) [;(n-1)! \leq n^ne^{-n}.e;]

Multiplicando por [;n;],

 [;n! \leq n^ne^{-n}.e.n;]

Extraindo a raíz enésima,

[;n!^{1/n} \leq ne^{-1}e^{1/n}n^{1/n};]

Dividindo por [;n;]

[;\rightarrow \left(\frac{n!}{n^n}\right)^{1/n} \leq \frac{1}{e}.e^{1/n}n^{1/n};] 

Então, por [;I;] e [;II;],  ficamos com uma desigualdade mais esclarecedora envolvendo [;\left(\frac{n!}{n^n}\right)^{1/n};]:
 [;\frac{1}{e}.e^{1/n} \leq \left(\frac{n!}{n^n}\right)^{1/n} \leq \frac{1}{e}.e^{1/n}n^{1/n};]

Vejam que, quando [;n \rightarrow \infty;], temos [;e^{1/n} \rightarrow 1;] e [;n^{1/n} \rightarrow 1;]. Logo,

[;\lim_{n \rightarrow \infty} \frac{1}{e}.e^{1/n} = \frac{1}{e};] ( pela esquerda)

[;\lim_{n \rightarrow \infty} \frac{1}{e}.e^{1/n}n^{1/n} = \frac{1}{e};] ( pela direita )
e, em consequência,
 
[;\lim_{n \rightarrow \infty} \left(\frac{n!}{n^n}\right)^{1/n} = \frac{1}{e};]


Referência bibliográfica:  Cálculo, Volume [;I;], de Serg Lang, Ed.Ao Livro Técnico SA, [;1969;].

4 comentários:

  1. Muito interessante os passos para obter o valor do limite. Na última equação não faltou elevar a 1/n?

    ResponderExcluir
  2. Oi, Paulo!

    Limites e somatórios infinitos são sempre interessantes pois sempre estão sob um manto de mistério.

    Aqui temos uma nova definição para [;e;] que é o inverso deste limite.

    E também vemos nela algo sobre a fórmula de Stirling.

    Obrigado pelo elogio e correção.

    ResponderExcluir
  3. Adorei o post. O problema eh bastante bonito. vlw pelo post.
    HUGOCITO

    ResponderExcluir
    Respostas
    1. Obrigado, Hugocito!

      É apenas uma aplicação de somas superiores e inferiores.

      Excluir